- Thu Nov 16, 2017 5:49 pm
#41672
Complete Question Explanation
(The complete setup for this game can be found here: lsat/viewtopic.php?t=4937)
The correct answer choice is (C)
Answer choice (A) is incorrect because O does not work on Wednesday. Answer choice (B) is incorrect because K works with N, but Thursday is not N’s first work day, a violation of the last rule. Answer choice (D) is incorrect because S works earlier than P’s first day, a violation of the fourth rule. Answer choice (E) is incorrect because N works, but N does not work at least two consecutive days. Thus, answer choice (C) is correct.
(The complete setup for this game can be found here: lsat/viewtopic.php?t=4937)
The correct answer choice is (C)
Answer choice (A) is incorrect because O does not work on Wednesday. Answer choice (B) is incorrect because K works with N, but Thursday is not N’s first work day, a violation of the last rule. Answer choice (D) is incorrect because S works earlier than P’s first day, a violation of the fourth rule. Answer choice (E) is incorrect because N works, but N does not work at least two consecutive days. Thus, answer choice (C) is correct.